What are the Quadratic Residues Modulo p?

  • Thread starter Thread starter knowLittle
  • Start date Start date
  • Tags Tags
    Quadratic
Click For Summary
SUMMARY

The discussion centers on the concept of quadratic residues modulo an odd prime p, specifically addressing the proof that there are exactly (p-1)/2 quadratic residues. Participants clarify that if b1 and b2 are distinct numbers such that b1² ≡ b2² (mod p), then it must follow that b1 = b2, as p cannot divide their difference unless they are equal. The argument hinges on the fact that the absolute difference |b1 - b2| is less than (p-1)/2, confirming that the only solution is b1 = b2. This leads to the conclusion that the squares of the integers from 1 to (p-1)/2 are all distinct modulo p.

PREREQUISITES
  • Understanding of modular arithmetic and congruences
  • Familiarity with the properties of prime numbers
  • Knowledge of quadratic residues and their definitions
  • Basic algebraic manipulation involving integers
NEXT STEPS
  • Study the properties of quadratic residues in number theory
  • Learn about the Legendre symbol and its applications
  • Explore the concept of primitive roots modulo p
  • Investigate the distribution of quadratic residues and non-residues
USEFUL FOR

Mathematicians, students studying number theory, and anyone interested in the properties of primes and modular arithmetic.

knowLittle
Messages
307
Reaction score
3

Homework Statement


Let p be an odd prime. A number ##a\in \mathbb{Z} _{p}^{\ast }## is a quadratic residue if the equation ##x^{2}=a\left( \mod p\right)## has a solution for the unknown x.

a. Show that there are exactly (p-1)/2 = quadratic residues, modulo p.

The Attempt at a Solution


I have access to the "proof", but I have some questions that need clarification:

Suppose that b1 and b2 are numbers between 1 and (p - 1)/2 , and suppose that
##b_{1}^{2}\equiv b_{2}^{2}(mod p) ## We want to show that b1 = b2. The fact that ##b_{1}^{2}\equiv b_{2}^{2}(modp)## means that p divides ##\left( b_{1}^{2}-b_{2}^{2}\right) =\left( b_{1}-b_{2}\right) \left( b_{1}+b_{2}\right)##
OR that ##\left( b_{1}^{2}-b_{2}^{2}\right) =\left( b_{1}-b_{2}\right) \left( b_{1}+b_{2}\right)## = p. k, where k is an integer.

However, b1+b2 is between 2 and p-1, so it can't be divisible by p.
Thus, p must divide b1-b2. But, |b1-b2| < (p-1)/2, so the only way for b1-b2 to be divisible by p is to have b1=b2.
QUESTION: p represents an odd prime. Primes can only be divisible by 1 or itself.
By stating that |b1-b2|< (p-1)/2 and that b1=b2, they try to explain that the b1 and b2 is not 0, but it's just the same number. Does it mean that b1=b2 has to be 1?
Or, is it saying that it's impossible that p divides b1^(2) - b2^(2) given b1≠b2 ?


This shows that ##1^{2},2^{2},\ldots ,\left( \dfrac {p-1} {2}\right) ^{2}## are all different modulo p, so there are exactly (p-1) /2 quadratic residues modulo p.


Thank you.
 
Physics news on Phys.org
knowLittle said:

Homework Statement


Let p be an odd prime. A number ##a\in \mathbb{Z} _{p}^{\ast }## is a quadratic residue if the equation ##x^{2}=a\left( \mod p\right)## has a solution for the unknown x.

a. Show that there are exactly (p-1)/2 = quadratic residues, modulo p.

The Attempt at a Solution


I have access to the "proof", but I have some questions that need clarification:

Suppose that b1 and b2 are numbers between 1 and (p - 1)/2 , and suppose that
##b_{1}^{2}\equiv b_{2}^{2}(mod p) ## We want to show that b1 = b2. The fact that ##b_{1}^{2}\equiv b_{2}^{2}(modp)## means that p divides ##\left( b_{1}^{2}-b_{2}^{2}\right) =\left( b_{1}-b_{2}\right) \left( b_{1}+b_{2}\right)##
OR that ##\left( b_{1}^{2}-b_{2}^{2}\right) =\left( b_{1}-b_{2}\right) \left( b_{1}+b_{2}\right)## = p. k, where k is an integer.

However, b1+b2 is between 2 and p-1, so it can't be divisible by p.
Thus, p must divide b1-b2. But, |b1-b2| < (p-1)/2, so the only way for b1-b2 to be divisible by p is to have b1=b2.
QUESTION: p represents an odd prime. Primes can only be divisible by 1 or itself.
By stating that |b1-b2|< (p-1)/2 and that b1=b2, they try to explain that the b1 and b2 is not 0, but it's just the same number. Does it mean that b1=b2 has to be 1?
Or, is it saying that it's impossible that p divides b1^(2) - b2^(2) given b1≠b2 ?


This shows that ##1^{2},2^{2},\ldots ,\left( \dfrac {p-1} {2}\right) ^{2}## are all different modulo p, so there are exactly (p-1) /2 quadratic residues modulo p.


Thank you.

They are trying to argue that b1 must equal b2. b1-b2=1 doesn't work. p has to divide b1-b2, not b1-b2 divide p.
 
Dick said:
They are trying to argue that b1 must equal b2. b1-b2=1 doesn't work. p has to divide b1-b2, not b1-b2 divide p.

I didn't say that b1-b2=1, I meant that b1=b2=1.
So, supposedly since p|(b1-b2); b1-b2= kp, where k is an integer. Are they saying that it's impossible p | (b1-b2) , unless b1=b2 or in other words that p|0 or 0= pk, where k is any integer and in this case k=0?
Or are they saying that it's impossible to have two different numbers b1, b2 such that p| (b1-b2)?
 
knowLittle said:
By stating that |b1-b2|< (p-1)/2 and that b1=b2, they try to explain that the b1 and b2 is not 0, but it's just the same number. Does it mean that b1=b2 has to be 1?
The argument is that if |c| < p and p divides c then c must be 0. Substituting c = b1-b2 shows p does not divide b1-b2 unless b1=b2, and substituting c = b1+b2 shows p does not divide that either.
 
knowLittle said:
I didn't say that b1-b2=1, I meant that b1=b2=1.
So, supposedly since p|(b1-b2); b1-b2= kp, where k is an integer. Are they saying that it's impossible p | (b1-b2) , unless b1=b2 or in other words that p|0 or 0= pk, where k is any integer and in this case k=0?
Or are they saying that it's impossible to have two different numbers b1, b2 such that p| (b1-b2)?

Yes, it's impossible because |b1-b2|<(p-1)/2. The only way it can be divisible by p is if b1-b2=0 so b1=b2.
 
Dick said:
Yes, it's impossible because |b1-b2|<(p-1)/2. The only way it can be divisible by p is if b1-b2=0 so b1=b2.

Thank you!
 
Question: A clock's minute hand has length 4 and its hour hand has length 3. What is the distance between the tips at the moment when it is increasing most rapidly?(Putnam Exam Question) Answer: Making assumption that both the hands moves at constant angular velocities, the answer is ## \sqrt{7} .## But don't you think this assumption is somewhat doubtful and wrong?

Similar threads

  • · Replies 12 ·
Replies
12
Views
2K
Replies
30
Views
3K
Replies
6
Views
3K
Replies
15
Views
4K
  • · Replies 4 ·
Replies
4
Views
2K
Replies
3
Views
1K
  • · Replies 1 ·
Replies
1
Views
2K
  • · Replies 9 ·
Replies
9
Views
2K
  • · Replies 13 ·
Replies
13
Views
4K